When Can This be a Perfect Square? | Turkish National Mathematical Olympiad 2009

Поділитися
Вставка
  • Опубліковано 5 сер 2022

КОМЕНТАРІ • 57

  • @mcwulf25
    @mcwulf25 Рік тому +42

    Another method, simpler.
    Move the 9 right and factorise both sides, where RHS starts as m^2.
    (p-2)p(p+2) = (m+3)(m-3)
    If p=2 then LHS is zero so m=3. One solution.
    If p>2 then LHS is product of 3 consecutive odd numbers, exactly one of which must be a multiple of 3. So RHS is a multiple of 3 so m is a multiple of 3. In which case we factor out 3 from both RHS factors. With m=3k:
    (p-2)p(p+2) = 9(k+1)(k-1)
    As the 9 must be absorbed by one of the LHS factors and the other RHS factors are consecutive odd numbers, there is a 1:1 mapping of the 3 factors each side.
    p is prime so one of the other LHS factors must equal 9. This gives us two more solutions:
    (2) p-2 = 9, so p=11, m=36
    (3) p+2 = 9, so p = 7, m=18.

    • @tianqilong8366
      @tianqilong8366 Рік тому

      but you need to show that why (p+2) and (p-2) cannot be a multiplier of 3, right?

    • @bait6652
      @bait6652 Рік тому +1

      No

    • @dqrk0
      @dqrk0 Рік тому +6

      @@tianqilong8366 its obvious, if p-2 is a multiple of 3, then (p-2)+3=p+1 is a multiple of 3 so p+2 cannot be and similarly for the other case

    • @wesleydeng71
      @wesleydeng71 Рік тому +1

      Nice. p=2 does not need to be a special case since 3 | (p-2)p(p+2) is always true.

    • @Szynkaa
      @Szynkaa Рік тому

      and what if k+1 or k-1 are also divisible by 3

  • @keithmasumoto9698
    @keithmasumoto9698 Рік тому +13

    125-20+9=114, so p=5 is also ruled out.

  • @mcwulf25
    @mcwulf25 Рік тому +5

    If anyone is interested, I can now prove my solution from a few days ago.
    It starts with saying that if p|(k-1) then mp= (k-1) and mp+2 = (k+1). Plug it all in and cancel p and form a quadratic in p. The discriminant is 81m^4 + 72m + 16 which must be a perfect square. This is always> m^4 for m>1 and if m>4 it is < (m^2+1)^2 so, as this is between squares of consecutive integers, no solutions for m>4. Test for m

  • @spiderjerusalem4009
    @spiderjerusalem4009 Рік тому +2

    also that p≡3 mod 4 only,
    because p³-4p+9 is even (for p>2)
    meaning either ≡ 0 or 2 (mod 4),
    but for it to be perfect square, then must be congruent to 0(mod4)
    p³+1 ≡ 0 (mod 4)
    p ≡ -1 ≡ 3(mod 4)

  • @pornhubxuxx
    @pornhubxuxx Рік тому

    Thanks for this amazing solution

    • @angelmendez-rivera351
      @angelmendez-rivera351 Рік тому

      How did you type your username on a keyboard?

    • @pornhubxuxx
      @pornhubxuxx Рік тому

      @@angelmendez-rivera351 copying from anyone's name in discord lol, copy & paste easy.

    • @prathikkannan3324
      @prathikkannan3324 Рік тому

      @@angelmendez-rivera351 Hello fellow Michael Penn and letsthinkcritically viewer. Just recently discovered this channel, its a gem!

  • @moeberry8226
    @moeberry8226 Рік тому +4

    You forgot to test p=5

  • @SuperYoonHo
    @SuperYoonHo Рік тому

    thanks!!!

  • @papanujian7758
    @papanujian7758 8 місяців тому

    wishing your new video, sir

  • @graham741
    @graham741 13 днів тому

    LHS must be even assuming it's not an even prime (but p=2 works too) so then RHS is odd, meaning it's congruent to 0 mod 4. Therefore p^3 + 1 = 0 mod 4 so p must be 3 mod 4, and primes must be 1 or 5 mod 6, so then p must be 11 or 7 mod 12. The only primes generated by this are 7,11,19,23,31, and 43 and checking yields p = 7,11 (it's annoying if you don't have a calculator but very doable with some mod checks + divisibility checks, eg if it's congruent to 0 modulo by n and not n^2 where n is not a perfect square then it cannot be a perfect square)

  • @mariomestre7490
    @mariomestre7490 Рік тому +2

    why change +-6k-4 , in 6k+-4? in 2:39. Merci

  • @iainfulton3781
    @iainfulton3781 Рік тому

    Turn on postifications

  • @lakshya5946
    @lakshya5946 Рік тому +2

    Hmm 🤔🧐 I'm also interested to see the answer 😌

  • @vvvppp777
    @vvvppp777 Рік тому +2

    why did you take mod p at the start?

    • @prathikkannan3324
      @prathikkannan3324 Рік тому

      It's a very common strategy. Once you see plenty of olympiad number theory problems, taking mod p is immediately obvious. In short, we like to simplify things, whether that means get some information on "x", create some bounds, or collapse into an easier equation to deal with, and reduction modulo n gives us these simplifications :)

  • @wonjonghyeon
    @wonjonghyeon Рік тому +2

    Where has the condition that p is greater than or equal to 5 gone?

    • @fix5072
      @fix5072 Рік тому +1

      He ruled out p=2, 3 first, then got a bound on all p>3 and then checked all possible p (2, 3, 5, 7, 11, 13)

    • @papanujian7758
      @papanujian7758 8 місяців тому

      oh i seee. thanks@@fix5072

  • @tianqilong8366
    @tianqilong8366 Рік тому +2

    Is it supposed to be +-6k + 4 instead of 6k +- 4 ??

  • @anjanavabiswas8835
    @anjanavabiswas8835 Рік тому

    I read the question as p^3 + 4p + 9 = perfect square. :(

  • @todddean7722
    @todddean7722 Рік тому +2

    I think you lost me at 3:30 to 3:50.

  • @nasrullahhusnan2289
    @nasrullahhusnan2289 Рік тому

    Trivial solutions are p=0 and p=2. As p is a prime number then p=2

  • @iainfulton3781
    @iainfulton3781 Рік тому

    You skipped p = 5 ffs

  • @padraiggluck2980
    @padraiggluck2980 Рік тому

    👍

  • @user-hq7hi2sl2o
    @user-hq7hi2sl2o Рік тому +1

    asnwer= 7 isi mom nag force isit no than 😅🥵🥶😂